jas770
Thanks Received: 0
Vinny Gambini
Vinny Gambini
 
Posts: 11
Joined: May 06th, 2010
 
 
 

Q10 - If the public library

by jas770 Mon Jul 12, 2010 3:51 pm

Could someone explain why "B" is the correct answer to question 10? It doesn't make sense how area would effect the overall population.
User avatar
 
ManhattanPrepLSAT2
Thanks Received: 311
Atticus Finch
Atticus Finch
 
Posts: 303
Joined: July 14th, 2009
 
This post thanked 1 time.
 
 

Re: Q10 - If the public library

by ManhattanPrepLSAT2 Mon Jul 12, 2010 6:32 pm

Notice the core relationship in this argument:

The author uses two premises...

More people live in central Glenwood than in central Redville

+

Generally, people will walk to a library only if it's close to their home

To validate a conclusion...

A library in central Glenwood would be within walking distance for more people than would a library in central Redville

For all Strengthen questions, we know going in that the evidence presented is not going to be sufficient to prove the conclusion reached. We want to make sure to consider this gap between evidence and conclusion carefully before moving on to the answer choices.

Why doesn't the evidence presented here validate the conclusion?

The author is assuming that because more people live in central Glenwood, more people will be within walking distance of a library in central Glenwood. Is this valid?

Let's imagine that Central Glenwood is Los Angeles, population 3 million, and Central Redville is Irvine, population 200,000. Does that mean any library in Los Angeles is going to have more people within walking distance than any library in Irvine? Not necessarily. The library in Los Angeles might happen to be in a commercial area that doesn't have a lot of homes, and the library in Irvine might be in the middle of it's most crowded neighborhood.

In order to use information about population to prove something about population density (people within a certain radius of a certain building), we need to have information about how big a space this population lives in. Answer choice (B), by giving us this information, helps us prove that Glenwood is more densely populated than Redville.
 
Sweetangel
Thanks Received: 0
Vinny Gambini
Vinny Gambini
 
Posts: 17
Joined: April 30th, 2014
 
 
 

Re: Q10 - If the public library

by Sweetangel Wed Apr 30, 2014 1:30 pm

For the secondary weaken question, how does B weaken the conclusion, "the library would then be within walking distance of a larger number of library users"? The argument isn't about how many people WILL actually access the library, but about how many COULD. I don't see how the library's increased potential accessibility is weakened by stating that less people would use the library if relocated (irrelevant?).
User avatar
 
tommywallach
Thanks Received: 468
Atticus Finch
Atticus Finch
 
Posts: 1041
Joined: August 11th, 2009
 
 
 

Re: Q10 - If the public library

by tommywallach Mon May 05, 2014 1:55 pm

Hey SweetAngel,

That's question 11, which has a separate thread.

-t
Tommy Wallach
Manhattan LSAT Instructor
twallach@manhattanprep.com
Image
User avatar
 
Mab6q
Thanks Received: 31
Atticus Finch
Atticus Finch
 
Posts: 290
Joined: June 30th, 2013
 
This post thanked 1 time.
 
 

Re: Q10 - If the public library

by Mab6q Wed Oct 15, 2014 7:00 pm

Great explanation by Mike above. Let me just add why the other answer choices are incorrect.

A. this is absolutely irrelevant.

C. is also irrelevant. The author says if it was moved.....

D. another irrelevant answer because we aren't concerned with that group of people.

E. Might be tempting because of how it's worded. However, all it tells us is that the people who are currently walking to the library are walking more than what is considered walking distance. However this is irrelevant because we are not necessary interested in the number of people walking but the number of users who are within walking distance.
"Just keep swimming"